Module 27: Sales

Overview

The law of sales governs contracts for the sale of goods. Since a sale of goods is involved, Article 2 of the Uniform Commercial Code (UCC) applies. A sale of goods under the UCC is the sale of tangible, moveable property. Be sure that when you are faced with a contract question to determine whether the contract involved the sale of goods; if so, then you will apply the UCC rules outlined in this module. If the contract involved something other than goods, then you want to use the common law rules outlined in Module 26.

A. Contracts for Sale of Goods

B. Forming the Contract for the Sale of Goods

C. Performance of the Sales Contract

D. Transfer of Title (Property Rights)

E. Risk of Loss

F. Product Liability

G. Remedies under Sales Law

H. Leases under UCC

I. Contracts for the International Sales of Goods (CISG)

Key Terms

Multiple-Choice Questions

Multiple-Choice Answers and Explanations

Simulations

Simulation Solutions

One of the areas tested in sales is product liability. When studying this area, you should pay particular attention to the different legal theories under which an injured party may recover. Realize that an injured party may recover under the legal theories of breach of warranty, negligence, and strict liability. It is important that you know the circumstances under which these theories may be used. Other areas that are often tested are warranties; disclaimers; risk of loss; and remedies, rights, and duties of the buyer and seller. You should understand that a binding contract may be present under the UCC if the parties intend to be bound, even though certain elements of a contract may be missing. These open terms will be filled by specific provisions of the UCC. The parties to a sale need not be merchants for the UCC to apply; however, some rules vary if merchants are involved in the sales contract.

As you study this area, note that it builds on much of the material under contracts in the previous module. Therefore, as you study this area you should review the contract law rules, especially those in the previous module that apply to the UCC. Before beginning the reading you should review the key terms at the end of the module.

A. Contracts for Sale of Goods

1. Article 2 of the Uniform Commercial Code, in general, controls contracts for the sale of goods for any dollar amount
a. “Goods” include tangible personal property (whether specially manufactured or not)
(1) Does not include sales of investment securities, accounts receivable, contract rights, copyrights, or patents

EXAMPLE
S sells B a stereo. The UCC applies.


EXAMPLE
S sells ome to B. The common law rules rather than the UCC rules apply to this contract since it involves the sale of real property.


EXAMPLE
F sells to M several bushels of wheat. The UCC applies to fungible goods also (i.e., goods in which one unit is considered the equivalent of the other units).

b. In general, UCC applies to sales and leases of hardware as well as to sales and licensing of software.
(1) However, if software is heavily customized based on services of consultant, common law applies.
c. Article 2 of UCC has been passed into law by every state (except Louisiana which has adopted only portions of UCC)
(1) Federal courts also use principles in Article 2 for sales of goods.
d. UCC applies whether sale is between merchants or consumers but some rules change if merchant involved

EXAMPLE
S sells his used refrigerator to B, a neighbor. The UCC applies to this transaction.

e. The UCC, unlike the common law, implies that a contract exists where it is the intent of the parties to create a contract, even though some technical element of contract may be missing
2. General concepts
a. Merchant—one who deals in the kind of goods being sold, or one who holds self out as having superior knowledge and skills as to the goods involved, or one who employs another who qualifies as a merchant
b. Good faith
(1) The UCC assume that parties entering into sales contracts will do so in good faith
(2) Good faith means that parties will be honest with each other
(3) Additionally for merchants it also means to observe reasonable commercial standards of fair dealing in the trade
c. UCC supplies missing terms for contracts
(1) Under common law if terms are missing there is no contract because the terms are not definite
(2) UCC fills in missing terms if parties have shown intent to enter into a contract
d. When terms of a sales contract are unclear the terms can be explained by
(1) Course of dealing: If parties have had previous dealings, we look to see how the unclear term was interpreted or acted upon by the parties in the past
(2) Usage of trade: How the term is typically used in the particular trade, business, or industry.

B. Forming the Contract for the Sale of Goods

1. The UCC adopts most of the common law rules for contracts
a. Assume the common law rule is the same as the UCC rule
b. What follows are the changes that the UCC makes to the common law rules
2. Offer
a. Offer under the UCC can be less definite
b. Generally the UCC permits:
(1) Open price terms: A reasonable/market price will be supplied if necessary.
(2) Open quantity terms
(a) Quantity term does need to be included for contracts over $500, under the Statute of Frauds
(b) Quantity term does not, however have to be precise
(c) Output contracts are permissible: Buyer promises to purchase all that a seller can produce
(d) Requirements contracts are permissible: Seller will provide the quantity of goods that the buyer needs
c. Firm offer rule
(1) Promise to hold an offer open, even if not supported by consideration, is irrevocable if the promise is
(a) In writing, and,
(b) Made by a merchant.
(2) If firm offer does not state specific time, it will remain open for reasonable time, based on the nature of the subject matter not to exceed three months

EXAMPLE
M, a merchant, agrees in a letter signed by M to sell B 1,000 widgets, with the offer to remain open for five weeks. Even if M tries to revoke this offer before the five-week period, B may still accept.


EXAMPLE
M, a merchant, agrees in signed writing to sell B 1,000 widgets, stating that the offer will remain open for 120 days. B accepts the offer on the 95th day. If nothing has occurred to terminate the offer prior to acceptance, offer and acceptance are present. The irrevocable nature of this offer would end after three months, but the offer would not automatically terminate. The offer would remain in existence for the stated period (120 days) unless terminated by other means.

3. Acceptance
a. UCC rejects the mirror image rule; instead the UCC focuses on the parties’ intent to be bound
b. The intent to be bound approach creates some problems

EXAMPLE
I offer to sell you 1,000 pens for $100 and ship them via UPS; you accept, but you want them shipped via Federal Express. Under the UCC there is a contract because we both agreed to the sale/purchase of the pens. But how should the goods be shipped: UPS or Federal Express? The answer is contained in the material below entitled Battle of forms.

c. Battle of forms—between merchants, additional terms included in the acceptance become part of the contract unless
(1) Original offer precludes such, or
(2) New terms materially alter the original offer, or
(3) The original offeror gives notice of his/her objection within a reasonable time

EXAMPLE
In the above example, if we are both merchants, then the additional/changed term you proposed: shipping via Federal Express would become part of the contract unless I make a timely objection.

(4) If the parties are not merchants, then the offeror needs to expressly accept the additional/changed term.
d. Accommodation
(1) When a seller ships substitute goods because the seller does not have the goods ordered by the buyer
(2) If the buyer accepts substituted goods, a contract exists
(3) If buyer refuses substituted goods, then there is no contract
(4) If seller provides notice to buyer that shipment is only an accommodation, seller’s actions are not viewed as an acceptance

EXAMPLE
Value Hardware placed an order with Odin Industries for 800 Thor hammers on June for delivery on August 15. On June 3, Odin sent Value a written acceptance. On August 12, Odin shipped 800 Asgard hammers because Odin was out of Thor hammers. Odin also stated on the invoice that the shipment was an accommodation. The hammers arrived at Value on August 15. Odin has breached the contract. Once a seller has accepted an order, the seller cannot simply substitute goods and call it an accommodation.


EXAMPLE
Assume Value placed the same order, on June 1 and stated, “I need the hammers as soon as possible.” Odin responded by shipping the 800 Asgard hammers on June 3 with a note stating that the shipment was an accommodation. If Value accepts, there is a contract. If Value does not accept, then Value can return the hammers and there is no contract.

4. a. Under the UCC, a contract may be modified without new consideration if done in good faith

EXAMPLE
B agrees in a contract to buy 300 electrical parts for $1.00 each from S. B later points out to S that he can get the same parts from D for $.90 each and asks for a price reduction. S reduces the price to $.90 each. This new contract is enforceable even though B gave no new consideration. Note that if S had required B to pay the $1.00 as originally agreed, B would be in breach of contract if he failed to go through with the original contract.

b. Common law requires new consideration on both sides for any modification

EXAMPLE
B agreed, in a written contract, to pay $10,000 to S for certain real estate. Later, B said he was having difficulty getting the $10,000 so S agreed to reduce the price to $9,000. S can still enforce the full $10,000 because B gave no new consideration for the modification.

5. Legality and Capacity are the same under the UCC and the common law
6. Under the UCC version of Statute of Frauds, contracts for sale of goods for $500 or more must be in writing or be evidenced by a writing.
a. A sufficient writing
(1) Must contain quantity term
(2) Signature of party to be charged
(3) Indicate the existence of a contract
b. If the contract is between merchants, then a written confirmation is sufficient to serve as a writing unless objection is made within 10 days of receiving the confirmation

EXAMPLE
Pat operates a fish market. Pat phoned Shelley, a fish wholesaler, to see if Shelley could sell Pat 200 pounds of sockeye salmon for $1,000 for delivery in two weeks. Shelley agreed and e-mailed Pat a written confirmation of Pat’s order. Pat did not respond to the e-mail. When the fish is delivered two weeks later is Pat obligated to take delivery? Yes, the contract falls under the UCC provision of the Statute of Frauds: Salmon is a good and the contract exceeds $500. Although the signature of the party to be charged, Pat, is not present here; Shelley’s written confirmation is sufficient since both parties are merchants.

NOTE: If Pat was not a merchant, then the writing is not sufficient to satisfy the Statute of Frauds and Pat could refuse delivery.


c. If contract is modified, must be in writing if after modification it is for $500 or more

EXAMPLE
B agrees in a contract to buy widgets from S for $500. Later, S agrees to a reduction in price to $490. The first contract must be in writing (absent any exceptions), but the modified contract may be oral.

d. Exceptions to writing requirement (these are important)
(1) Oral contract involving specially manufactured goods (i.e., not saleable in ordinary course of business) if seller has made substantial start in their manufacture (or even made a contract for necessary raw materials) is enforceable
(2) Oral contract is enforceable against party who admits it in court, but not beyond quality of goods admitted
(3) Goods that have been paid for (if seller accepts payment) or goods which buyer has accepted are part of enforceable contract even if oral

EXAMPLE
B orally agrees to purchase 10,000 parts from S for $1 each. B later gives S $6,000 for a portion of the parts. S accepts the money. In absence of a written agreement, B may enforce a contract for 6,000 parts but not for the full 10,000 parts.

C. Performance of the Sales Contract

1. Seller’s performance
a. Tender: A party is ready, willing, and able to perform the obligations of the contract.
b. Place of tender (performance) is the seller’s place of business, unless contract provides otherwise
(1) Parties may choose place of tender by contract
(2) This is one of the UCC’s “gap” filling provisions: It only applies if the parties have not accounted for the place of tender in the contract
c. Perfect tender rule
(1) Seller’s tender of goods must conform exactly, to the terms of the contract
(2) If Seller’s tender is not perfect, buyer may:
(a) Reject the whole lot of goods
(b) Accept the whole lot of goods
(c) Accept some units and reject the remaining units

EXAMPLE
Wonka ordered 5,000 orange gaskets from Acme. The gaskets are used in various machines that Wonka uses to produce candy. Acme timely delivered 5,000 gaskets in five crates of 1,000 gaskets each. When Wonka opened the crates he saw that the gaskets were yellow instead of orange. Even if the color has no affect on how the gaskets work, Wonka may still reject the lot. If Wonka needed 1,500 gaskets immediately, Wonka could accept two crates, or 2,000 gaskets. Wonka cannot, however, split a shipping unit, a crate, so if he needs 1,500 gaskets, he will need to accept two crates.

(3) Exceptions
(a) Agreement by parties that seller did not need to perfectly tender
(b) Seller cures (fixes) the defect
(c) Installment contracts: Buyer may only reject a defective installment, not entire contract

EXAMPLE
Wonka orders 5,000 orange gaskets from Acme to be delivered in boxes of 500 each month for the next 10 months. Acme perfectly tendered for the first three months, but in the fourth month Acme sent yellow gaskets instead. Wonka may reject only the shipment in the fourth month.

2. Buyer’s performance
a. Payment is due at the time of delivery
b. Buyer has a right to inspect the goods prior to payment; except for good that were shipped COD (Cash on delivery)
c. Buyer should inspect goods at time of delivery to preserve buyer’s remedies against a seller who has shipped defective or nonconforming goods
d. Buyer may reject good if goods are defective
(1) Buyer must disclose nature of the defect to seller. (Gives seller opportunity to cure defect)
(2) Buyer must either return goods or hold goods with reasonable care
e. Buyer may accept goods
(1) Buyer may revoke prior acceptance if
(a) Seller did not provide promised cure, or
(b) Goods contained hidden defect: A hidden defect is a defect that the buyer could not perceive with reasonable inspection at the time of delivery
3. Excused performance: Either buyer or seller’s obligation to perform may be excused if any of the following occur
a. Destruction of the goods prior to performance
b. Commercial impracticality: Performance has become excessively expensive or difficult due to unforeseeable events
c. Assurance of performance
(1) If there is a reasonable basis to believe that the other party will not perform as promised, the uncertain party may
(a) Make a written demand for adequate assurance of performance
(b) A response assuring performance must be provided within a reasonable time, not exceeding 30 days.
(c) Failure to respond in a timely fashion is evidence of anticipatory repudiation
d. Anticipatory repudiation: One party clearly indicates that it is unwilling, or unable, to perform the contract as promised
e. Cooperation necessary for performance was not provided

EXAMPLE
Wonka ordered 5,000 orange gaskets from Acme. Wonka promised to provide Acme with a specific orange dye. Despite repeated requests from Acme, Wonka never provided the dye. Acme’s performance of coloring the gadgets orange is excused.

f. Performance is not excused when substitute performance is possible

EXAMPLE
Wonka ordered 5,000 orange gaskets from Acme. Acme agreed to ship the gaskets by truck to Wonka’s factory. At the time of delivery there was a trucker’s strike. The strike does not excuse Acme’s performance. Acme could have the gaskets delivered by train or some other method of transportation.

g. Consignment—Arrangement in which agent (consignee) is appointed by consignor to sell goods if all the following conditions are met:
(1) Consignor keeps title to goods,
(2) Consignee is not obligated to buy or pay for goods,
(3) Consignee receives a commission upon sale, and
(4) Consignor receives proceeds of sale.
h. Document of title—Any document that in the regular course of business is accepted as adequate evidence that the person in possession of the document is entitled to receive, hold, and dispose of the document and the goods it covers
i. Bill of lading—A document of title that is issued by a private or common carrier in exchange for goods delivered to it for shipment. It may be negotiable or nonnegotiable.
j. Warehouse receipt—A document of title issued by a person engaged in the business of storing goods (i.e., a warehouseman). It acknowledges receipt of the goods, describes the goods stored, and contains the terms of the storage contract. It may be negotiable or nonnegotiable.

D. Transfer of Title (Property Rights)

1. Transfer of title to third parties
a. These are situations where a person acquires the property and then tries to transfer ownership of the acquired property to another person (a third party)
b. If party having voidable title transfers goods to a good-faith purchaser for value, the latter obtains good title
(1) Examples in which there is voidable title
(a) Goods paid for with a check subsequently dishonored
(b) Goods obtained by fraud, mistake, duress, or undue influence
(c) Goods obtained from minor
(d) Thieves and/or finders of property have void title

EXAMPLE
B buys a stereo from S but the check bounces. P, a good-faith purchaser, pays B for the stereo. S cannot get the stereo from P but must recover money from B.


EXAMPLE
Same as above except that B stole the stereo. P does not obtain title of the stereo. S can recover the stereo from P. This is true even if P was unaware that the stereo was stolen.

c. If a person entrusts possession of goods to a merchant who deals in those goods, a good-faith purchaser for value obtains title to these goods, unless s/he knew that this merchant did not own the goods.

EXAMPLE
C leaves his watch at a shop for repairs. The shop mistakenly sells the watch to B who is unaware of C’s interest. C cannot force B to turn over the watch because B now has title. Of course, C can recover monetary damages from the shop.

2. Transfer of title from a seller to a buyer
a. Once goods exist and are identified to the contract, the parties may agree as to when title passes
(1) Sale cannot take place until goods exist and have been identified to the contract
(a) Identification—occurs when the goods that are going to be used to perform the contract are shipped, marked or otherwise designated as such
(b) Identification gives buyer
[1] An insurable interest in the goods once they are identified to contract
[2] Right to demand goods upon offering full contract price once other conditions are satisfied
b. Otherwise, title generally passes when the seller completes his/her performance with respect to physical delivery
(1) If a destination contract, title passes on tender at destination (i.e., buyer’s place of business)
(2) If a shipping (point) contract, title passes when seller puts goods in the possession of the carrier
c. If seller has no duty to move the goods
(1) Title passes upon delivery of documents of title

EXAMPLE
Delivery of negotiable or nonnegotiable warehouse receipt passes title to buyer.

(2) If no document of title exists, title passes at the time and place of contracting if the goods are identifiable
(3) If goods not identified, there is only a contract to sell; no title passes
d. Rejection (justified or not) of goods or a justified revocation of acceptance by buyer reverts title to seller
e. Taking a security interest is irrelevant to passage of title

E. Risk of Loss

1. Risk of loss is independent of title under UCC, but rules regarding the transfer of both are similar
2. General rules when seller ships goods that conform to the contract
a. Parties may agree as to which party bears risk of loss or has title; otherwise UCC rules below apply
b. Shipment contract
(1) Risk of loss transfers to buyer when the goods are delivered to the common carrier
(2) Generally designated as “FOB seller’s warehouse,” but any indication of the seller’s place of business is sufficient

EXAMPLE
Seller is in San Francisco and buyer is in Chicago: FOB San Francisco.

(a) Other shipment contract designations
[1] CIF: Cost, insurance, and freight are included in price
[2] C & F: Shipping contract in which cost and freight are included in price
c. In international sales shipment contracts under United Nations Convention for the International Sale of Goods, risk of loss passes to buyer upon delivery to first carrier for transmission to buyer.
(1) This can be modified by agreement.
d. Destination contract
(1) Risk of loss transfer to buyer when the goods reach their intended destination and are tendered to the buyer
(2) Usually designated as “FOB buyer’s place of business”
e. If no shipping terms are specified, then the presumption is a shipment contract
f. Trial sales: Seller is allowing buyer to try the good prior to the actual sale
(1) Sale on approval: Goods may be returned even if they conform to the contract
(a) Goods not considered sold until buyer approves or accepts as sale
(b) Seller retains title and risk of loss until buyer accepts goods
(c) Creditors of buyer cannot reach goods until buyer accepts
(2) Sale or return: Goods may be returned even if they conform to the contract
(a) Goods bought for use or resale
(b) Sale is final if goods not returned during period specified
(c) Buyer obtains risk of loss and title until goods are returned
(d) Creditors of buyer can reach the goods while in buyer’s possession, unless notice of seller’s interest is posted or filed as required
g. If risk of loss is not covered by above rules, then
(1) Merchant sellers transfer the risk of loss when buyer takes physical possession of goods
(2) Nonmerchant sellers transfer risk of loss to buyer upon tender

EXAMPLE
Max agreed to purchase a used drum set from Buddy. Buddy is not in the business of selling musical instruments. Buddy tendered delivery to Max, after receiving Max’s check in the mail. Max told Buddy that he would stop by the following week and pick up the drum set. The next night, after a huge storm, Buddy’s basement flooded, and ruined the drum set. Who bears the loss of the drum set? Max will bear the loss because the risk transferred to Max upon tender. Note: If Buddy had been a merchant, Buddy would bear the loss.

h. If goods are held in warehouse and seller has no right to move them, risk of loss passes to buyer
(1) Upon proper negotiation of a negotiable document of title
(2) Within a reasonable time after delivery of a nonnegotiable document of title
(3) Once warehouseman acknowledges buyer’s right to goods if no document of title
3. Sales involving breach of contract
a. If seller breaches (typically seller has shipped nonconforming goods)
(1) Risk of loss remains with seller until cure by seller or acceptance by buyer to extent of buyer’s deficiency in insurance coverage
(2) Title passes under original terms despite delivery of nonconforming goods

EXAMPLE
Wonka orders 5,000 orange gaskets from Acme. The contract states that the gaskets will be shipped FOB Acme’s warehouse. While the gaskets are in transit to Wonka’s warehouse, the gaskets are destroyed when the common carrier transporting the gaskets is involved in an accident. A review of the common carrier’s bill of lading reveals that Acme had shipped 5,000 yellow gaskets, instead of orange gaskets.
Analysis: Title to the gaskets passed to Wonka because this is a shipment contract. Normally, the shipment contract would also place the risk of loss upon Wonka as well. However, since Acme shipped nonconforming gaskets (yellow instead of orange), the risk of loss never transferred to Wonka, so Acme bears the loss here.

b. If buyer breaches, risk of loss passes to buyer to extent of seller’s deficiency in insurance for a commercially reasonable time.
4. Risk of loss can be covered by insurance. In general, party has an insurable interest whenever s/he can suffer damage.
a. Buyer usually allowed an insurable interest when goods are identified to the contract
b. Seller usually has an insurable interest so long as s/he has title or a security interest
F. Product Liability—a manufacturer or seller may be responsible when a product is defective and causes injury or damage to a person or property. There are three theories under which manufacturers and sellers may be held liable. (In each fact pattern, consider all three, although proof of any one creates liability.)
1. Warranty Liability—purchaser of a product may sue based on the warranties made
a. Warranty of title
(1) Seller warrants good title, rightful transfer, and freedom from any security interest or lien of which the buyer has no knowledge

EXAMPLE
A seller of stolen goods would be liable to a buyer for damages since the seller has no title.

(2) Merchant warrants goods to be free of rightful claim of infringement (e.g., patent or trademark), unless buyer furnished specifications to seller for manufacture of the goods
(3) Can only be disclaimed by specific language or circumstances that give buyer reason to know s/he is receiving less than full title
(a) Cannot be disclaimed by language such as “as is”
b. Express warranties (may be written or oral)
(1) Any affirmation of fact or promise made by the seller to the buyer that relates to the goods and becomes part of the basis of the bargain creates an express warranty that the goods shall conform to the affirmation or promise
(a) Sales talk, puffing, or a statement purporting to be merely the seller’s opinion does not create a warranty
(b) Must form part of the basis of bargain
[1] Would include advertisements read by buyer
[2] Normally would not include warranties given after the sale or contract was made
(c) No intent to create warranty is needed on the part of the seller
(d) Seller or buyer may be merchant or consumer
(2) Any description of the goods which is made part of the basis of the bargain creates an express warranty that the goods shall conform to the description
(3) Any sample or model that is made part of the basis of the bargain creates an express warranty that the goods shall conform to the sample or model
(4) It is not necessary to the creation of an express warranty that the seller use formal words such as “warranty” or “guarantee”
c. Implied warranties
(1) Warranty of merchantability—goods are fit for ordinary purpose
(a) This warranty also guarantees that goods are properly packaged and labeled
(b) This warranty applies if
[1] Seller must be a merchant with respect to goods of the kind being sold, and
[2] Warranty is not modified or excluded
[3] Then if goods not fit for ordinary use, breach of this warranty occurs
(2) Warranty of fitness for a particular purpose
(a) Created when the seller knows of the particular use for which the goods are required and further knows that the buyer is relying on skill and judgment of seller to select and furnish suitable goods for this particular use

EXAMPLE
A buyer relies upon a paint salesperson to select a particular exterior house paint that will effectively cover existing siding.

(b) Buyer must actually rely on seller’s skill and judgment
(c) Product is then warranted for the particular expressed purpose and seller may be liable if the product fails to so perform
(d) Applicable both to merchants and nonmerchants
d. UCC, being consumer oriented, allows these warranties to extend to parties other than the purchaser even without privity of contract (contractual connection between parties)
(1) Extends to a buyer’s family and also to guests who may reasonably be expected to use and/or be affected by the goods and who are injured

EXAMPLE
A dinner guest breaks a tooth on a small piece of metal in the food. Note that in food, the substance causing injury normally must be foreign, not something customarily found in it (bone in fish).

e. Disclaimers—warranty liability may be escaped or modified by disclaimers (also available at common law without rules defining limits of disclaimers)
(1) A disclaimer of merchantability can be written or oral but must use the word “merchantability” unless implied warranties are disclaimed as in (3) below
(2) To disclaim the implied warranty of fitness for a particular purpose, the disclaimer must be in writing and conspicuous
(3) Both the warranty of merchantability and fitness for a particular purpose can be disclaimed by oral or written language such as “as is” or “with all faults”
(4) Written disclaimers must be clear and conspicuous
(5) If the buyer has had ample opportunity to inspect the goods or sample, there is no implied warranty as to any defects which ought reasonably to have been discovered
(6) Implied warranties may be excluded or modified by course of dealings, course of performance, or usage of trade
(7) A disclaimer inconsistent with an express warranty is not effective (i.e., a description of a warranty in a contract cannot be disclaimed)
(8) Limitations on consequential damages for personal injuries are presumed to be unconscionable and thus unenforceable if on consumer goods
2. Negligence
a. Must prove the following elements
(1) Duty of manufacturer to exercise reasonable (due) care to the injured party
(a) Consider likelihood of harm, seriousness of harm
(b) May be based on violation of statute but this is not necessary
(2) Breach of duty of reasonable care
(a) Insufficient instructions may cause breach of duty
(b) Did the manufacturer/seller fail to act as a reasonable manufacturer/seller?
(3) Manufacturer’s failure to exercise reasonable care caused the injury/damages.
(a) Direct cause: In general, if injury would not have happened without defendant’s conduct, there is cause in fact
(b) Proximate cause: Negligence set into motion an unbroken chain of events which led to injury/damages
(4) Injury/damages: The plaintiff must be able to show that s/he has suffered some recognizable loss
b. Privity of contract is not needed because suit not based on contract

EXAMPLE
A car manufacturer is negligent in the manufacture and design of brakes and as a result, a driver is severely injured. The driver may sue the manufacturer even if he bought the car from a retailer.


EXAMPLE
In the example above, even a pedestrian injured because of the brake problem may recover from the manufacturer.

c. Defenses to negligence
(1) Contributory negligence
(a) That is, plaintiff helped cause accident
(b) Complete bar to recovery
(c) Most states instead use comparative negligence in which damages are allocated between plaintiff and defendant based on relative fault
(2) Assumption of risk
3. Strict product liability
a. Manufacturers, sellers, and lessors who normally deal in this type of product are liable to users of products without proof of fault or lack of reasonable care if following other elements are proven
(1) Product was defective when sold
(a) Based on poor design, inadequate warnings, improper assembly, or unsafe materials
(2) Defect is unreasonably dangerous to user
(a) Based on normal expectations
(3) Product reaches user without significant changes
(4) Defect caused the injury
b. Defense of acting with reasonable care, contributory negligence, comparative negligence, disclaimer or lack of privity is unavailable
(1) Assumption of risk and misuse are defenses

EXAMPLE
Herb is injured while lifting up his power lawnmower to trim his hedges. Manufacturer would not be liable since product was not being used for intended purpose.

4. American Law Institute has published its Restatement (Third) of Torts: Product Liability
a. This is significant development and many courts now cite it
b. However, treat it as a minority rule which has not superceded important long-standing rules of negligence
c. Restatement (Third)’s product liability rule basically states those in business of selling or distributing defective products are liable for harm to individuals or property resulting from such defect
(1) Rule covers not only manufacturers but other sellers engaged in business of selling such products that caused harm to plaintiffs
(2) Note that Restatement (Third) does not require products to be unreasonably dangerous
d. Restatement (Third) adds to general rule above by establishing three types of product defects
(1) Manufacturing defects
(a) Take place when product does not conform with its intended use when leaves manufacturer

EXAMPLE
Flawed products, damaged products, or products assembled incorrectly.

(2) Inadequate warnings and instructions
(a) Manufacturers and sellers liable for failing to warn or instruct about reasonably foreseeable harms
[1] Need not warn of generally known or obvious risks
(3) Design defects
(a) A product has a design defect if its foreseeable risk of harm could have been avoided or reduced by different design

NOW REVIEW MULTIPLE-CHOICE QUESTIONS 1 THROUGH 35

G. Remedies under Sales Law

1. In general, either party may, upon breach by other, cancel the contract and terminate executory obligations
a. Unlike common law rescission, however, cancellation does not discharge a claim for damages
2. Seller’s remedies
a. Seller has right to cure nonconformity (i.e., tender conforming goods)
(1) Within original time of contract, or
(2) Within reasonable time if seller thought nonconforming tender would be acceptable
(3) Seller must notify buyer of his intention to cure
b. Seller may resell goods if buyer breaches before acceptance
(1) May be public or private sale
(a) If private, must give notice to buyer who breached; otherwise, losses cannot be recovered
(b) If seller resells in a commercially reasonable manner, s/he may recover any loss on the sale from the buyer who breached, but s/he is not responsible to buyer who breached for profits made on resale
(c) In any event, good-faith purchasers take free of original buyer’s claims
c. If seller learns that buyer is insolvent and buyer does not have the document of title, seller may stop delivery of goods in carrier’s possession unless buyer pays cash
d. Seller may recover goods received by an insolvent buyer if demand is made within ten days of receipt
(1) However, if the buyer has made a written misrepresentation of solvency within three months before delivery, this ten-day limitation does not apply
(2) If buyer is insolvent, seller may demand cash to make delivery
e. Seller may recover damages
(1) If buyer repudiates agreement or refuses goods, seller may recover the difference between market price at time of tender and contract price, plus incidental damages, minus expenses saved due to buyer’s breach
(2) If the measure of damages stated above in (1) is inadequate to place the seller in as good a position as performance would have, then the seller can sue for the lost profits, plus incidental damages, less expenses saved due to the buyer’s breach
(a) Loss profits are consequential damages and as such are recoverable when foreseeable by breaching party
(3) The seller can recover the full contract price when
(a) The buyer has already accepted the goods
(b) Conforming goods have been destroyed after the risk of loss had transferred to buyer
(c) The seller is unable to resell the identified goods
f. Under Uniform Computer Information Transactions Act (UCITA), licensor of its software has special self-help remedies available to protect its software, the right to be paid for usage, or its trade secrets.
(1) Licensor may use bugs, etc., that disable that software from further misuse if certain requirements are met
(a) Licensee must have specifically agreed to that self-help remedy
(b) Licensor must give licensee at least fifteen days’ notice before using that remedy as well as who licensee can contact about any questions
(c) Licensor is not permitted to use this remedy if there is a significant risk of personal injury or public safety, or if there is a significant risk to information of other parties
3. Buyer’s remedies
a. Buyer may reject nonconforming goods, either in entirety or any commercial unit (e.g., bale, carload, etc.)
(1) Buyer has right to inspect goods before acceptance or payment
(a) Must do so in reasonable time and give notice to seller (failure may operate as acceptance)
(b) Buyer must have reasonable time to inspect
(2) Buyer must care for goods until returned
(3) If buyer is a merchant, s/he must follow reasonable instructions of seller (e.g., ship, sell)
(a) Right to indemnity for costs
(4) If goods are perishable or threatened with decline in value, buyer must make reasonable effort to sell
(5) Buyer has a security interest in any goods in his/her possession to the extent of any payments made to seller and any expenses incurred
(a) S/he may sell the goods to recover payments
b. Under Uniform Computer Information Transactions Act (UCITA), consumers who make electronic errors while ordering have special rights, if consumer, upon learning of error, does following
(1) Immediately notifies seller that s/he made error (as soon as s/he learns of error)
(2) Buyer does not use or benefit from information, software or products ordered
(3) Delivers all copies to seller or destroys them at seller’s request
(4) Buyer in error pays all costs of processing and shipping to seller
(5) Note that nonconsumer buyer may not use these more favorable provisions of this Act

EXAMPLE
Buyer intends to purchase one copy of a DVD from ABC Company. The buyer, who is purchasing this DVD for consumer use, mistakenly orders ten copies from ABC’s website. The buyer is protected by following the steps given above.

c. Buyer may recover damages measured by the difference between the contract price and the market value of the goods at the time buyer learns of the breach, plus any incidental damages and consequential damages
(1) Consequential damages are damages resulting from buyer’s needs that the seller was aware of at the time of contracting
(2) Consequential damages cannot be recovered if buyer could reasonably have prevented these (mitigation of damages)
d. Buyer has the right of cover
(1) Buyer can buy substitute goods from another seller—buyer will still have the right to damages after engaging in “cover”
(a) Damages are difference between cost of cover and contract price, plus incidental and consequential damages
(b) Failure to cover does not bar other remedies
e. Once goods to the contract have been identified, buyer obtains rights in those goods
(1) Identification occurs when goods under contract are
(a) Shipped
(b) Marked as part of the contract, or
(c) In some way designated as part of contract
(2) Buyer obtains
(a) Insurable interest in those goods, and
(b) Right to obtain goods, called replevin, upon offering contract price
[1] Replevin (suing for possession of the goods wrongfully held by the seller) is not allowed if buyer can cover
f. Buyer may obtain specific performance if goods are unique or in other proper circumstances even if goods are not identified to the contract
(1) Proper circumstances may exist when other remedies (such as monetary damages or remedy of cover) are inadequate

EXAMPLE
S agrees to sell B an antique car of which only one exists. If S later refuses to go through with the contract, B may require S to sell him the unique car under the remedy of specific performance.

4. Statute of limitations for sale of goods is four years
a. An action for breach must be commenced within this period
b. Parties may agree to reduce to not less than one year but may not extend it
c. Statute of limitations begins running when the contract is breached

NOW REVIEW MULTIPLE-CHOICE QUESTIONS 36 THROUGH 48

H. Leases under UCC

1. Law governing leases has been slow to develop and has been “tacked on” for various other areas of law such as property law and secured transactions
2. Now Article 2A of the UCC applies to any transaction creating a lease regardless of form
3. Article 2A is now law in majority of states
4. Article 2A is quite lengthy, but for purpose of CPA exam, note that its provisions are similar to Article 2 except that Article 2A applies to leases and Article 2 applies to sales of goods
5. Under Article 2A
a. Lessor is person who transfers right to possess named goods and to use them in described ways by lessee
6. Note the following provisions where Article 2A is similar to Article 2:
a. Statute of frauds except that stated minimum is $1,000 instead of $500 that applies to sales of goods
(1) There are three exceptions to Statute of Frauds whereby leases need not be in writing even if for $1000 or more (note that these are similar to three exceptions to Statute of Frauds for sales of goods)
(a) Specially manufactured goods when goods are not suitable for sale or lease in the ordinary course of lessor’s business
(b) Lessor or lessee admits to oral lease in court proceedings
[1] Only enforceable up to quantity admitted
(c) Part acceptance in which lease is enforceable up to amount accepted by lessee

EXAMPLE
E leases under an oral agreement 900 personal computers. A, the lessor, ships 400 of the personal computers to E. After accepting the 400, E decides she does not want to lease the other 500. E is liable for the lease of the 400 personal computers under the part acceptance exception even though the agreement was oral. She would be liable for the lease of the full 900 personal computers if the agreement had been for less than $1,000 which is not the case here.

b. Rules on acceptance, revocation of acceptance, and rejection of goods
c. Remedies are similar to sellers’ and buyers’ remedies including the important concept of cure
d. Principles for performance include anticipatory repudiation or breach, (including use of adequate assurance to avoid a breach), and the concept of impracticability
e. Leases may be assigned
f. Use general principles of contract and sales law for these
(1) Warranties
(2) Parol evidence
(3) Firm offers
(4) Risk of loss rules
(5) Concept of unconscionable agreements
g. Provision for sublease by lessee
h. Leased goods may become fixtures
i. Lessor has right to take possession of leased property after default without requirement of court adjudication
7. Leases under Article 2A of UCC may be in any manner sufficient to show by words or conduct that lessor and lessee intended to form a lease of identified goods
8. Finance lease is three-party transaction in which lessor acquires title or right to possess goods from supplier
a. Lessor does not manufacture or supply goods for lessee but third-party supplier does according to lease agreement

I. Contracts for the International Sales of Goods (CISG)

1. Contracts for sales of goods between persons or companies of different countries follow the important rules of CISG
2. Many provisions of CISG are similar to UCC provisions but differences are handled under CISG because USA has this treaty with many countries in South America, Central America, North America, and most countries in Europe
a. By Constitutional Law, CISG has priority over UCC when it applies and when it conflicts with UCC
b. The following are important areas where CISG and UCC are different
(1) Price terms
(a) May be left open under UCC, in which case UCC provides that price is reasonable price at time of delivery
(b) CISG requires that price term be included for there to be a contract
[1] CISG allows exception if method to determine price in future is clearly specified in contract
(2) Time contract formed
(a) Unlike UCC, CISG specifies that contract is formed only when acceptance is received by offeror
(b) Also under CISG, acceptance happens at moment requested act is performed, not at the time notice is given of acceptance to offeror
(3) Acceptances
(a) CISG provides that there is no contract if terms in acceptance are different from terms in offer
[1] Acceptance is effective if differences are not material
[a] However, almost ever term in contract under CISG is considered material
(4) Irrevocable offers
(a) UCC allows offers that are not supported by consideration to be irrevocable if they are written and also meet certain other criteria
(b) CISG allows offeror to make offer irrevocable by orally stating so
(5) Written contracts
(a) UCC has $500 rule for sales of goods
(b) CISG provides that sales contracts may be oral with no rule regarding amount of money
[1] Also provides that proof of contract can be by any reasonable means
(6) Parties are encouraged to have choice-of-language and choice-of-law clauses in contracts to help settle any disputes

KEY TERMS

Destination contract. An agreement that generally transfers title and risk of loss of the goods to the buyer when the goods reach their destination, usually the buyer’s place of business.

Firm offer. An offer that is irrevocable despite the lack of consideration from the offeree to hold the offer open. Offer must be in writing and made by a merchant to be firm.

Good faith. Parties will operate honestly in the course of the transaction; think of trying to honor the spirit of the agreement more than the literal agreement.

Goods. Moveable personal property

Implied warranty. A guarantee that automatically exists, unless it is disclaimed by the seller. Some examples include the warranty of title, the implied warranty of merchantability, and the implied warranty of fitness for a particular purpose.

Merchant. A person who regularly buys or sells the goods that are involved in the contract.

Perfect tender rule. The goods that the seller delivers must conform exactly to the terms of the contract.

Risk of loss. Parties to the contract allocate which party will bear the loss if the goods are damaged or destroyed.

Shipment contract. An agreement that generally provides that title and risk of loss pass to the buyer when the seller delivers the goods to the common carrier.

Strict product liability. Sellers are held responsible for injuries that their goods cause, even if the seller exercised due care.

Tender. When a party to the contract is ready, willing, and able to perform the promise of the contract.

Transfer of title. Ownership of goods is transferred from the seller to the buyer.

Warranty. A guarantee concerning the quality, performance, or other characteristic of the good.

Multiple-Choice Questions (1–48)

A. Contracts for Sale of Goods

1. Under the Sales Article of the UCC, when a written offer has been made without specifying a means of acceptance but providing that the offer will only remain open for ten days, which of the following statements represent(s) a valid acceptance of the offer?

I. An acceptance sent by regular mail the day before the ten-day period expires that reaches the offeror on the eleventh day.

II. An acceptance faxed the day before the ten-day period expires that reaches the offeror on the eleventh day, due to a malfunction of the offeror’s printer.

a. I only.

b. II only.

c. Both I and II.

d. Neither I nor II.

2. Under the Sales Article of the UCC, a firm offer will be created only if the

a. Offer states the time period during which it will remain open.

b. Offer is made by a merchant in a signed writing.

c. Offeree gives some form of consideration.

d. Offeree is a merchant.

3. On May 2, Mason orally contracted with Acme Appliances to buy for $480 a washer and dryer for household use. Mason and the Acme salesperson agreed that delivery would be made on July 2. On May 5, Mason telephoned Acme and requested that the delivery date be moved to June 2. The Acme salesperson agreed with this request. On June 2, Acme failed to deliver the washer and dryer to Mason because of an inventory shortage. Acme advised Mason that it would deliver the appliances on July 2 as originally agreed. Mason believes that Acme has breached its agreement with Mason. Acme contends that its agreement to deliver on June 2 was not binding. Acme’s contention is

a. Correct, because Mason is not a merchant and was buying the appliances for household use.

b. Correct, because the agreement to change the delivery date was not in writing.

c. Incorrect, because the agreement to change the delivery date was binding.

d. Incorrect, because Acme’s agreement to change the delivery date is a firm offer that cannot be withdrawn by Acme.

4. Under the Sales Article of the UCC, which of the following statements is correct?

a. The obligations of the parties to the contract must be performed in good faith.

b. Merchants and nonmerchants are treated alike.

c. The contract must involve the sale of goods for a price of more than $500.

d. None of the provisions of the UCC may be disclaimed by agreement.

5. Which of the following contracts is handled under common law rules rather than under Article 2 of the Uniform Commercial Code?

a. Oral contract to have hair styled in which expensive products will be used on the hair.

b. Oral contract to purchase a textbook for $100.

c. Written contract to purchase an old handcrafted chair for $600 from a private party.

d. Written contract to purchase a heater from a dealer to be installed by the buyer in her home.

6. Cookie Co. offered to sell Distrib Markets 20,000 pounds of cookies at $1.00 per pound, subject to certain specified terms for delivery. Distrib replied in writing as follows:

We accept your offer for 20,000 pounds of cookies at $1.00 per pound, weighing scale to have valid city certificate.

Under the UCC

a. A contract was formed between the parties.

b. A contract will be formed only if Cookie agrees to the weighing scale requirement.

c. No contract was formed because Distrib included the weighing scale requirement in its reply.

d. No contract was formed because Distrib’s reply was a counteroffer.

7. EG Door Co., a manufacturer of custom exterior doors, verbally contracted with Art Contractors to design and build a $2,000 custom door for a house that Art was restoring. After EG had completed substantial work on the door, Art advised EG that the house had been destroyed by fire and Art was canceling the contract. EG finished the door and shipped it to Art. Art refused to accept delivery. Art contends that the contract cannot be enforced because it violated the Statute of Frauds by not being in writing. Under the Sales Article of the UCC, is Art’s contention correct?

a. Yes, because the contract was not in writing.

b. Yes, because the contract cannot be fully performed due to the fire.

c. No, because the goods were specially manufactured for Art and cannot be resold in EG’s regular course of business.

d. No, because the cancellation of the contract was not made in writing.

8. On May 2, Handy Hardware sent Ram Industries a signed purchase order that stated, in part, as follows:

Ship for May 8 delivery 300 Model A-X socket sets at current dealer price. Terms 2/10/net 30.

Ram received Handy’s purchase order on May 4. On May 5, Ram discovered that it had only 200 Model A-X socket sets and 100 Model W-Z socket sets in stock. Ram shipped the Model A-X and Model W-Z sets to Handy without any explanation concerning the shipment. The socket sets were received by Handy on May 8.

Which of the following statements concerning the shipment is correct?

a. Ram’s shipment is an acceptance of Handy’s offer.

b. Ram’s shipment is a counteroffer.

c. Handy’s order must be accepted by Ram in writing before Ram ships the socket sets.

d. Handy’s order can only be accepted by Ram shipping conforming goods.

9. Under the UCC Sales Article, which of the following conditions will prevent the formation of an enforceable sale of goods contract?

a. Open price.

b. Open delivery.

c. Open quantity.

d. Open acceptance.

10. Webstar Corp. orally agreed to sell Northco, Inc. a computer for $20,000. Northco sent a signed purchase order to Webstar confirming the agreement. Webstar received the purchase order and did not respond. Webstar refused to deliver the computer to Northco, claiming that the purchase order did not satisfy the UCC Statute of Frauds because it was not signed by Webstar. Northco sells computers to the general public and Webstar is a computer wholesaler. Under the UCC Sales Article, Webstar’s position is

a. Incorrect because it failed to object to Northco’s purchase order.

b. Incorrect because only the buyer in a sale-of-goods transaction must sign the contract.

c. Correct because it was the party against whom enforcement of the contract is being sought.

d. Correct because the purchase price of the computer exceeded $500.

11. Patch, a frequent shopper at Soon-Shop Stores, received a rain check for an advertised sale item after Soon-Shop’s supply of the product ran out. The rain check was in writing and stated that the item would be offered to the customer at the advertised sale price for an unspecified period of time. A Soon-Shop employee signed the rain check. When Patch returned to the store one month later to purchase the item, the store refused to honor the rain check. Under the Sales Article of the UCC, will Patch win a suit to enforce the rain check?

a. No, because one month is too long a period of time for a rain check to be effective.

b. No, because the rain check did not state the effective time period necessary to keep the offer open.

c. Yes, because Soon-Shop is required to have sufficient supplies of the sale item to satisfy all customers.

d. Yes, because the rain check met the requirements of a merchant’s firm offer even though no effective time period was stated.

12. A sheep rancher agreed in writing to sell all the wool shorn during the shearing season to a weaver. The contract failed to establish the price and a minimum quantity of wool. After the shearing season, the rancher refused to deliver the wool. The weaver sued the rancher for breach of contract. Under the Sales Article of the UCC, will the weaver win?

a. Yes, because this was an output contract.

b. Yes, because both price and quantity terms were omitted.

c. No, because quantity cannot be omitted for a contract to be enforceable.

d. No, because the omission of price and quantity terms prevents the formation of a contract.

13. Under the Sales Article of the UCC, the warranty of title

a. Provides that the seller cannot disclaim the warranty if the sale is made to a bona fide purchaser for value.

b. Provides that the seller deliver the goods free from any lien of which the buyer lacked knowledge when the contract was made.

c. Applies only if it is in writing and assigned by the seller.

d. Applies only if the seller is a merchant.

14. Under the Sales Article of the UCC, most goods sold by merchants are covered by certain warranties. An example of an express warranty would be a warranty of

a. Usage of trade.

b. Fitness for a particular purpose.

c. Merchantability.

d. Conformity of goods to sample.

15. Under the Sales Article of the UCC, which of the following statements is correct regarding the warranty of merchantability arising when there has been a sale of goods by a merchant seller?

a. The warranty must be in writing.

b. The warranty arises when the buyer relies on the seller’s skill in selecting the goods purchased.

c. The warranty cannot be disclaimed.

d. The warranty arises as a matter of law when the seller ordinarily sells the goods purchased.

16. On May 2, Handy Hardware sent Ram Industries a signed purchase order that stated, in part, as follows:

Ship for May 8 delivery 300 Model A-X socket sets at current dealer price. Terms 2/10/net 30.

Ram received Handy’s purchase order on May 4. On May 5, Ram discovered that it had only 200 Model A-X socket sets and 100 Model W-Z socket sets in stock. Ram shipped the Model A-X and Model W-Z sets to Handy without any explanation concerning the shipment. The socket sets were received by Handy on May 8.

Assuming a contract exists between Handy and Ram, which of the following implied warranties would result?

I. Implied warranty of merchantability.

II. Implied warranty of fitness for a particular purpose.

III. Implied warranty of title.

a. I only.

b. III only.

c. I and III only.

d. I, II, and III.

17. Under the UCC Sales Article, an action for breach of the implied warranty of merchantability by a party who sustains personal injuries may be successful against the seller of the product only when

a. The seller is a merchant of the product involved.

b. An action based on negligence can also be successfully maintained.

c. The injured party is in privity of contract with the seller.

d. An action based on strict liability in tort can also be successfully maintained.

18. Which of the following conditions must be met for an implied warranty of fitness for a particular purpose to arise in connection with a sale of goods?

I. The warranty must be in writing.

II. The seller must know that the buyer was relying on the seller in selecting the goods.

a. I only.

b. II only.

c. Both I and II.

d. Neither I nor II.

19. Under the UCC Sales Article, the implied warranty of merchantability

a. May be disclaimed by a seller’s oral statement that mentions merchantability.

b. Arises only in contracts involving a merchant seller and a merchant buyer.

c. Is breached if the goods are not fit for all purposes for which the buyer intends to use the goods.

d. Must be part of the basis of the bargain to be binding on the seller.

20. Cook Company, a common carrier trucking company, made a contract to transport some video equipment for Jackson Company. Cook is trying to limit its liability in the contract. In which of the following situations can Cook not avoid liability?

I. In transit, the driver of Cook’s truck damages the video equipment when the driver causes an accident.

II. An unknown thief steals the video equipment while in transit. Cook committed no negligence in this theft.

III. The video equipment is destroyed when a bridge under the truck collapses because of an earthquake.

a. I only.

b. I and II only.

c. I, II, and III.

d. I and III only.

21. High sues the manufacturer, wholesaler, and retailer for bodily injuries caused by a power saw High purchased. Which of the following statements is correct under strict liability theory?

a. Contributory negligence on High’s part will always be a bar to recovery.

b. The manufacturer will avoid liability if it can show it followed the custom of the industry.

c. Privity will be a bar to recovery insofar as the wholesaler is concerned if the wholesaler did not have a reasonable opportunity to inspect.

d. High may recover even if he cannot show any negligence was involved.

22. To establish a cause of action based on strict liability in tort for personal injuries that result from the use of a defective product, one of the elements the injured party must prove is that the seller

a. Was aware of the defect in the product.

b. Sold the product to the injured party.

c. Failed to exercise due care.

d. Sold the product in a defective condition.

23. A common carrier bailee generally would avoid liability for loss of goods entrusted to its care if the goods are

a. Stolen by an unknown person.

b. Negligently destroyed by an employee.

c. Destroyed by the derailment of the train carrying them due to railroad employee negligence.

d. Improperly packed by the party shipping them.

24. McGraw purchased an antique rocking chair from Tillis by check. The check was dishonored by the bank due to insufficient funds. In the meantime, McGraw sold the rocking chair to Rio who had no knowledge that McGraw’s check had been dishonored. Which of the following is correct?

a. Tillis may repossess the rocking chair from Rio.

b. Tillis may recover money damages from Rio.

c. Tillis may recover money damages from McGraw.

d. Tillis may recover damages from McGraw based on fraud.

25. Yancie took her bike in to Pete’s Bike Sales and Repair to have it repaired. Pete said he would need to have her leave it for two days. The next day, one of Pete’s employees sold Yancie’s bike to Jake. Jake paid for the bike with a credit card, unaware that Pete did not own the bike. Which of the following is correct?

a. Yancie can repossess the bike from Jake if she pays Jake. Yancie then recovers the price from Pete.

b. Pete can repossess the bike from Jake and then return it to Yancie.

c. Yancie can sue Jake for monetary damages only.

d. Jake has title to the bike.

26. Under the Sales Article of the UCC, unless a contract provides otherwise, before title to goods can pass from a seller to a buyer, the goods must be

a. Tendered to the buyer.

b. Identified to the contract.

c. Accepted by the buyer.

d. Paid for.

27. Under the Sales Article of the UCC, in an FOB place of shipment contract, the risk of loss passes to the buyer when the goods

a. Are identified to the contract.

b. Are placed on the seller’s loading dock.

c. Are delivered to the carrier.

d. Reach the buyer’s loading dock.

28. On May 2, Lace Corp., an appliance wholesaler, offered to sell appliances worth $3,000 to Parco, Inc., a household appliances retailer. The offer was signed by Lace’s president, and provided that it would not be withdrawn before June 1. It also included the shipping terms: “FOB Parco’s warehouse.” On May 29, Parco mailed an acceptance of Lace’s offer. Lace received the acceptance June 2.

If Lace inadvertently ships the wrong appliances to Parco and Parco rejects them two days after receipt, title to the goods will

a. Pass to Parco when they are identified to the contract.

b. Pass to Parco when they are shipped.

c. Remain with Parco until the goods are returned to Lace.

d. Revert to Lace when they are rejected by Parco.

29. Under the Sales Article of the UCC and the United Nations Convention for the International Sale of Goods (CISG), absent specific terms in an international sales shipment contract, when will risk of loss pass to the buyer?

a. When the goods are delivered to the first carrier for transmission to the buyer.

b. When the goods are tendered to the buyer.

c. At the conclusion of the execution of the contract.

d. At the time the goods are identified to the contract.

30. Which of the following statements applies to a sale on approval under the UCC Sales Article?

a. Both the buyer and seller must be merchants.

b. The buyer must be purchasing the goods for resale.

c. Risk of loss for the goods passes to the buyer when the goods are accepted after the trial period.

d. Title to the goods passes to the buyer on delivery of the goods to the buyer.

31. Under the Sales Article of UCC, which of the following events will result in the risk of loss passing from a merchant seller to a buyer?

  Tender of the goods at the seller’s place of business Use of the seller’s truck todeliver the goods
a. Yes Yes
b. Yes No
c. No Yes
d. No No

32. Cey Corp. entered into a contract to sell parts to Deck, Ltd. The contract provided that the goods would be shipped “FOB Cey’s warehouse.” Cey shipped parts different from those specified in the contract. Deck rejected the parts. A few hours after Deck informed Cey that the parts were rejected, they were destroyed by fire in Deck’s warehouse. Cey believed that the parts were conforming to the contract. Which of the following statements is correct?

a. Regardless of whether the parts were conforming, Deck will bear the loss because the contract was a shipment contract.

b. If the parts were nonconforming, Deck had the right to reject them, but the risk of loss remains with Deck until Cey takes possession of the parts.

c. If the parts were conforming, risk of loss does not pass to Deck until a reasonable period of time after they are delivered to Deck.

d. If the parts were nonconforming, Cey will bear the risk of loss, even though the contract was a shipment contract.

33. Under the Sales Article of the UCC, which of the following factors is most important in determining who bears the risk of loss in a sale of goods contract?

a. The method of shipping the goods.

b. The contract’s shipping terms.

c. Title to the goods.

d. How the goods were lost.

34. Bond purchased a painting from Wool, who is not in the business of selling art. Wool tendered delivery of the painting after receiving payment in full from Bond. Bond informed Wool that Bond would be unable to take possession of the painting until later that day. Thieves stole the painting before Bond returned. The risk of loss

a. Passed to Bond at Wool’s tender of delivery.

b. Passed to Bond at the time the contract was formed and payment was made.

c. Remained with Wool, because the parties agreed on a later time of delivery.

d. Remained with Wool, because Bond had not yet received the painting.

35. Funston, a retailer, shipped goods worth $600 to a customer by using a common carrier. The contract used by the common carrier, and agreed to by Funston, limited liability to $100 unless a higher fee is paid. Funston did not pay the higher fee. The goods were shipped FOB destination point and were destroyed in transit due to a flash flood. Which of the following is correct?

a. Funston will suffer a loss of $500.

b. Funston will suffer a loss of $600.

c. Funston’s customer will suffer a loss of $500.

d. Funston’s customer will suffer a loss of $600.

G. Remedies

36. Under the Sales Article of the UCC, which of the following statements regarding liquidated damages is (are) correct?

I. The injured party may collect any amount of liquidated damages provided for in the contract.

II. The liquidated damage provision cannot be excessive.

a. I only.

b. II only.

c. Both I and II.

d. Neither I nor II.

37. Under the Sales Article of the UCC, and unless otherwise agreed to, the seller’s obligation to the buyer is to

a. Deliver the goods to the buyer’s place of business.

b. Hold conforming goods and give the buyer whatever notification is reasonably necessary to enable the buyer to take delivery.

c. Deliver all goods called for in the contract to a common carrier.

d. Set aside conforming goods for inspection by the buyer before delivery.

38. Under the Sales Article of the UCC, which of the following rights is (are) available to a seller when a buyer materially breaches a sales contract?

  Right to cancel the contract Right to recover damages
a. Yes Yes
b. Yes No
c. No Yes
d. No No

39. Under the Sales Article of the UCC, the remedies available to a seller when a buyer breaches a contract for the sale of goods may include

  The right to resell goods identified to the contract The right to stop a carrier from delivering the goods
a. Yes Yes
b. Yes No
c. No Yes
d. No No

40. Lazur Corp. entered into a contract with Baker Suppliers, Inc. to purchase a used word processor from Baker. Lazur is engaged in the business of selling new and used word processors to the general public. The contract required Baker to ship the goods to Lazur by common carrier pursuant to the following provision in the contract: “FOB Baker Suppliers, Inc. loading dock.” Baker also represented in the contract that the word processor had been used for only ten hours by its previous owner. The contract included the provision that the word processor was being sold “as is” and this provision was in a larger and different type style than the remainder of the contract.

Assume that Lazur refused to accept the word processor even though it was in all respects conforming to the contract and that the contract is otherwise silent. Under the UCC Sales Article,

a. Baker can successfully sue for specific performance and make Lazur accept and pay for the word processor.

b. Baker may resell the word processor to another buyer.

c. Baker must sue for the difference between the market value of the word processor and the contract price plus its incidental damages.

d. Baker cannot successfully sue for consequential damages unless it attempts to resell the word processor.

41. On February 15, Mazur Corp. contracted to sell 1,000 bushels of wheat to Good Bread, Inc. at $6.00 per bushel with delivery to be made on June 23. On June 1, Good advised Mazur that it would not accept or pay for the wheat. On June 2, Mazur sold the wheat to another customer at the market price of $5.00 per bushel. Mazur had advised Good that it intended to resell the wheat. Which of the following statements is correct?

a. Mazur can successfully sue Good for the difference between the resale price and the contract price.

b. Mazur can resell the wheat only after June 23.

c. Good can retract its anticipatory breach at any time before June 23.

d. Good can successfully sue Mazur for specific performance.

42. Pickens agreed to sell Crocket 100 cases of napkins with the name of Crocket’s restaurant on the napkins. In the enforceable contract, it was specified that delivery will take place on April 15, which is one month after Pickens and Crocket signed the contract. Crocket wanted the napkins by April 15 because the grand opening of the restaurant was scheduled for April 17. On April 11, Pickens tells Crocket that he has too many orders and will not be able to deliver the napkins. What options does Crocket have?

I. Treat it as a present breach of contract and cancel the contract.

II. Wait for a reasonable time to see if Pickens will deliver.

a. I only.

b. II only.

c. Either I or II.

d. Neither I nor II.

43. Under the Sales Article of the UCC, which of the following rights is (are) available to the buyer when a seller commits an anticipatory breach of contract?

image

44. Larch Corp. manufactured and sold Oak a stove. The sale documents included a disclaimer of warranty for personal injury. The stove was defective. It exploded causing serious injuries to Oak’s spouse. Larch was notified one week after the explosion. Under the UCC Sales Article, which of the following statements concerning Larch’s liability for personal injury to Oak’s spouse would be correct?

a. Larch cannot be liable because of a lack of privity with Oak’s spouse.

b. Larch will not be liable because of a failure to give proper notice.

c. Larch will be liable because the disclaimer was not a disclaimer of all liability.

d. Larch will be liable because liability for personal injury cannot be disclaimed.

45. Under the Sales Article of the UCC, which of the following events will release the buyer from all its obligations under a sales contract?

a. Destruction of the goods after risk of loss passed to the buyer.

b. Impracticability of delivery under the terms of the contract.

c. Anticipatory repudiation by the buyer that is retracted before the seller cancels the contract.

d. Refusal of the seller to give written assurance of performance when reasonably demanded by the buyer.

46. Rowe Corp. purchased goods from Stair Co. that were shipped COD. Under the Sales Article of the UCC, which of the following rights does Rowe have?

a. The right to inspect the goods before paying.

b. The right to possession of the goods before paying.

c. The right to reject nonconforming goods.

d. The right to delay payment for a reasonable period of time.

47. Under the UCC Sales Article, a plaintiff who proves fraud in the formation of a contract may

a. Elect to rescind the contract and need not return the consideration received from the other party.

b. Be entitled to rescind the contract and sue for damages resulting from the fraud.

c. Be entitled to punitive damages, provided physical injuries resulted from the fraud.

d. Rescind the contract even if there was no reliance on the fraudulent statement.

48. Sklar, CPA, purchased from Wiz Corp. two computers. Sklar discovered material defects in the computers ten months after taking delivery. Three years after discovering the defects, Sklar commenced an action for breach of warranty against Wiz. Wiz has raised the statute of limitations as a defense. The original contract between Wiz and Sklar contained a conspicuous clause providing that the statute of limitations for breach of warranty actions would be limited to eighteen months. Under the circumstances, Sklar will

a. Win because the action was commenced within the four-year period as measured from the date of delivery.

b. Win because the action was commenced within the four-year period as measured from the time he discovered the breach or should have discovered the breach.

c. Lose because the clause providing that the statute of limitations would be limited to eighteen months is enforceable.

d. Lose because the statute of limitations is three years from the date of delivery with respect to written contracts.

Multiple-Choice Answers and Explanations

Answers

image

Explanations

1. (c) Under the Sales Article of the UCC, acceptance is valid when sent if a reasonable method is used; therefore answer(c) is correct as both acceptances were sent prior to the end of the ten-day period.

2. (b) A firm offer is a written, signed offer concerning the sale of goods, by a merchant, giving assurance that it will be held open for a specified time and is irrevocable for that period, not to exceed three months. Answer(a) is incorrect because if the firm offer does not state a period of time, it will remain open for a reasonable period of time, not to exceed three months. Answer(c) is incorrect as consideration is not required for a firm offer, but for an option contract. Answer(d) is incorrect because under the firm offer rule, only the offeror need be a merchant.

3. (c) Under the UCC, an oral modification of an existing contract for the sale of goods for a price less than $500 is considered binding. Answer (b) is incorrect since the washer and dryer Mason contracted to buy cost less than $500, Acme’s oral agreement to change the date of delivery would be enforceable. Answer (a) is incorrect: The fact that Mason is not a merchant won’t affect whether the oral modification is binding. Answer (d) is incorrect: In order to have a firm offer, the offer must be made by a merchant in a signed writing which gives assurance that the offer will be held open. In this situation, the modification of an offer already accepted is being discussed rather than a firm offer.

4. (a) Under the Sales Article of the UCC, both the seller and buyer are obligated to perform a contract in good faith. Answer(b) is incorrect because certain provisions, such as the battle of forms provision, only apply to merchants. Answer(c) is incorrect because the Sales Article of the UCC applies to the sale of goods without regard to the price of goods. Answer(d) is incorrect because certain provisions of the UCC may be disclaimed by written or oral agreement, such as warranty liability.

5. (a) Article 2 of the UCC applies to sales of goods. Common law generally applies to contracts for services and real estate. Even though goods are used in this service contract, the predominate feature of this contract is the service. Answers (b), (c), and (d) are incorrect because each is a contract for the sale of goods. Additionally, for answer (b) Article 2 of the UCC governs this contract even though it is oral and for a small sum; for answer (c), even though the chair at one time involved a lot of labor, it is still a sale of goods; and for answer (d), whether the parties are merchants is not an issue on whether Article 2 applies. The heater which is not yet installed in the home is a sale of goods. Once it is installed in the home, it becomes part of the real estate for any future sale of the home. Common law rules would apply to any such future sale.

6. (a) Under common law, an acceptance must be unequivocal and unqualified in agreeing to the precise terms specified by the offer. However, the Uniform Commercial Code alters this general rule as far as the sales of goods is concerned. Under the UCC, an acceptance containing additional terms is a valid acceptance unless the acceptance is expressly conditional upon the offeror’s agreement to the additional terms. In this situation, a valid contract has been formed between Cookie Co. and Distrib Markets. Answer (b) is incorrect because Distrib Markets’ acceptance was not conditional upon Cookie’s agreement to the additional term and, thus, a contract is formed regardless of Cookie’s agreement or objection to the additional term. Answers (c) and (d) are incorrect because this contract was for the sale of goods and is governed by the UCC rather than by common law. Under common law, Distrib Markets’ reply would have been a rejection and counteroffer; but under the UCC, a contract was formed.

7. (c) This exception for specially manufactured goods, even if the contract is for over $500, is one of the important exceptions found in the Statute of Fraud provisions of the Uniform Commercial Code. Answer (a) is incorrect because the exception for specially manufactured goods applies to this fact pattern and thus this contract need not be in writing. Answer (b) is incorrect because the fire did not prevent the custom door contract from being performed. Answer (d) is incorrect because the contract was fully enforceable and Art had no legal right to cancel the contract.

8. (a) Ram may accept the offer by shipping the goods. Under the UCC, shipping nonconforming goods constitutes an acceptance, also unless the seller notifies the buyer that the shipment is given only as an accommodation to the buyer. Answer(b) is incorrect because this shipment counts as an acceptance, not as a counteroffer. Answer(c) is incorrect because an order to buy goods for prompt shipment allows the seller to accept by either a prompt promise to ship or by the actual prompt shipment itself. Answer (d) is incorrect because shipping nonconforming goods constitutes an acceptance under the UCC.

9. (d) In order to have a contract, there must be both an offer and an acceptance. Even though an acceptance can occur in different ways, by speech, by writing, or by action, the actual acceptance is a required element of a contract. Under the UCC Sales Article, a binding contract may be present if the parties had intended to form a contract even though certain elements of the contract are missing. These open terms will be filled by specific provisions of the UCC, including provisions for open price, open delivery, or open quantity. Note that in the case of quantity, output contracts, requirements contracts, and exclusive dealing’s contracts are enforceable though the actual quantity may not be known in advance.

10. (a) The UCC provides that a confirmation satisfies the UCC Statute of Frauds, if an oral contract between merchants is confirmed in writing within a reasonable period of time, and the confirmation is signed by the party sending it and received by the other party. Both parties are bound unless the party receiving the confirmation submits a written objection within ten days of receipt. In this situation, a valid contract has been formed since Webstar did not object to Northco’s purchase order. In a sale-of-goods transaction, the contract must be signed by the party to be charged to be enforceable. However, in the case of a written confirmation of an oral agreement between merchants, the confirmation need only be signed by the party sending the confirmation. The use of a signed purchase order satisfies the UCC Statute of Frauds.

11. (d) A firm offer is an offer for the sale of goods that is written and signed by a merchant (or employee of the merchant) that agrees to keep the offer open. This offer is valid without consideration for three months since no time was specified in the fact pattern. Patch will win in a suit to enforce the rain check because Patch tried to use it one month later. Answer (a) is incorrect because the UCC specifies a three-month period when no time is detailed in the firm offer. Answer (b) is incorrect because when no time is specified, the UCC gives Patch three months to accept the offer. Answer (c) is incorrect because there was no offer and acceptance when Patch first tried to purchase the advertised item.

12. (a) An output contract is enforceable under the UCC even though an actual quantity is not mentioned in the contract. The output contract is supported by consideration because the seller has agreed not to sell that output to any other party. Answer (b) is incorrect because when the price is omitted, the UCC construes it as the reasonable price at the time of delivery. The quantity is construed as the output of the sheep rancher. Answer (c) is incorrect because although quantity is an important term in the contract, the UCC allows the quantity term to be defined by output. Answer (d) is incorrect because the UCC allows price terms to be based on the reasonable price and quantity terms to be defined by output.

13. (b) Under the warranty of title, the seller warrants good title, rightful transfer and freedom from any security interest or lien of which the buyer has no knowledge at the time of sale. Answer(a) is incorrect because the warranty of title can be disclaimed by specific language or circumstances which give the buyer reason to know s/he is receiving less than full title. Answer(c) is incorrect because the warranty does not have to be in writing. Answer(d) is incorrect because the seller does not have to be a merchant for the seller to give the warranty of title.

14. (d) In the Sales Article of the UCC, express warranties include warranties that the goods will conform to any description used or any sample or model shown. Answer(a) is incorrect because although usage of trade can help interpret terms used in contracts, it is not a warranty. Answers(b) and(c) are incorrect because the warranty of fitness for a particular purpose and the warranty of merchantability are both implied warranties.

15. (d) The implied warranty of merchantability, which guarantees that goods are fit for ordinary purposes, arises as a matter of law when the seller is a merchant who ordinarily sells the goods purchased. Answer(a) is incorrect because the warranty is implied, and therefore need not be in writing. Answer (b) is incorrect because no reliance by the purchaser is necessary for the implied warranty of merchantability; the reliance described in answer (b) is a requirement for the implied warranty of fitness for a particular purpose. Answer(c) is incorrect because the warranty applies unless specifically disclaimed by the merchant.

16. (c) The implied warranty of merchantability is always implied if the seller is a merchant with respect to the type of goods being sold. Since Ram is a merchant, this warranty would apply. Also, under the UCC, the seller warrants good title, rightful transfer, and freedom from any security interest or lien of which the buyer has no knowledge when the contract was made. This warranty of title applies unless the merchant specifically disclaims it. In this situation, both the implied warranty of merchantability and the implied warranty of title apply. The implied warranty of fitness for a particular purpose is created only when a seller has reason to know the buyer’s particular purpose and knows the buyer is relying on the skill and judgment of the seller selecting the goods, which is not present here.

17. (a) The implied warranty of merchantability applies only when the seller is a merchant with respect to the type of goods being sold. The seller must be a merchant in order for the buyer to successfully sue under this warranty. Answer(b) is incorrect because the buyer does not have to prove negligence to be able to recover under this implied warranty. Answer(c) is incorrect because the implied warranty of merchantability extends to parties other than the purchaser even without privity of contract. Answer(d) is incorrect because an action for a breach based on the warranty of merchantability would not depend on the outcome of an action based on strict liability.

18. (b) The implied warranty of fitness for a particular purpose is created when a seller (merchant or nonmerchant) has reason to know the buyer’s particular purpose and knows the buyer is relying on the skill and judgment of the seller selecting the goods. Since the warranty of fitness for a particular purpose is an implied warranty, there is no requirement that it be made in writing.

19. (a) The implied warranty of merchantability may be disclaimed by a seller’s oral or written statement. This statement normally must contain some form of the word “merchantability” to be effective. However, goods sold “as is” or “with all faults” are an exception to that rule. Answer(b) is incorrect because the implied warranty of merchantability arises whenever the seller is a merchant with respect to the goods being sold. The status of the buyer is irrelevant. Answer(c) is incorrect because the implied warranty of merchantability guarantees that the goods are of an average fair quality and are fit for ordinary purposes. Under this warranty, the seller does not guarantee that the goods are fit for all purposes for which the buyer intends to use the goods. Answer(d) is incorrect because this warranty is always implied if the seller is a merchant. It does not have to be a part of the basis of the bargain to be binding on the seller.

20. (b) Common carriers’ liability is based on strict liability. As such, the common carrier is liable for losses to property whether or not the common carrier was negligent. Common law exceptions to strict liability include natural disasters which are responsible for damages.

21. (d) Under the theory of strict liability, the plaintiff must establish the following: (1)the seller was engaged in the business of selling the product, (2)the product was defective, (3)the defect was unreasonably dangerous to the plaintiff, and (4)the defect caused injury to the plaintiff. If the plaintiff can prove these elements, then the seller will be liable regardless of whether the seller was negligent or at fault for the defect. Thus, High can recover even if he cannot show any negligence was involved. Answer(a) is incorrect because contributory negligence is not an available defense in a strict liability case. Answer(b) is incorrect because the manufacturer’s only defenses are misuse and assumption of risk by the buyer. The fact that the manufacturer followed the custom of the industry is irrelevant under strict liability. Answer(c) is incorrect because privity of contract is not a defense under strict liability since the suit is not based on contract law.

22. (d) Under the theory of strict liability, the plaintiff must establish the following: (1)the seller was engaged in the business of selling the product, (2)the product was defective when sold, (3)the defect was unreasonably dangerous to the plaintiff, and (4) the defect caused injury to the plaintiff. If the plaintiff can prove these elements, then the seller will be liable regardless of whether the seller was negligent or at fault for the defect.

23. (d) The standard of care required for a common carrier bailee is based on strict liability rather than reasonable care. Common carrier bailees, however, are not liable for acts of God, acts of the shipper, or acts of a public enemy. In this case, the improper packing was done by the party doing the shipping. Answer(a) is incorrect because acts or theft by other parties make the common carrier liable. Answer(b) is incorrect because acts such as negligence, by others, still leave the common carrier liable. Answer(c) is incorrect because acts of a railroad employee cause the common carrier to be liable.

24. (c) Since Rio was a good-faith purchaser, Rio obtains good title to the rocking chair. Therefore, the remedy that Tillis has left is to sue McGraw for money damages. There are insufficient facts to show fraud, since the facts do not mention whether McGraw knew that the check would be dishonored when he wrote it.

25. (d) If a person entrusts possession of goods to a merchant who normally deals in that type of goods, a good-faith purchaser obtains title to those goods. Jake purchased the bike as he was unaware that Pete did not own the bike. As a good-faith purchaser, he obtains title to the bike. Answer(a) is incorrect because Yancie cannot repossess the bike from Jake because Jake obtained good title to the bike. Yancie can, however, get the value of the bike from Pete. Answer(b) is incorrect because Jake obtains title to the bike and, thus, Pete cannot repossess it from him. Answer(c) is incorrect because Yancie can recover the value of the bike from Pete, not Jake.

26. (b) A requirement needed for the title of goods to pass to the buyer is that the goods must have been identified to the contract. Answers(a) and(c) are incorrect because the seller can keep possession of goods and identify them to the contract and still have title pass to the buyer. Answer(d) is incorrect because title passes to the buyer based upon the terms of the agreement. Payment can take place before or after.

27. (c) In an FOB place of shipment contract, the buyer obtains the risk of loss once the goods are delivered to the carrier.

28. (d) The title of goods generally passes to the buyer when the seller completes performance with respect to the physical delivery of the goods. Because the shipping terms of the contract are FOB Parco’s warehouse, the title of goods passes to Parco on tender at the destination. This is true even if the goods are nonconforming. However, Parco’s rejection of the appliances will revert the title of the goods back to Lace at the time of the rejection.

29. (a) Under the Sales Article of the Uniform Commercial Code and the United Nations Convention for the International Sale of Goods, generally the risk of loss of the goods sold will pass to the buyer when the seller delivers goods to the first carrier for transmission to the buyer. Answers(b), (c), and(d) are incorrect because these would result in risk of loss to the buyer only if the contract specifically stated so, thus changing the general rule.

30. (c) The purchase of goods on a sale on approval allows the buyer to return the goods even if they conform to the contract. Therefore, the seller retains the title and the risk of loss until the buyer accepts the goods.

31. (d) Risk of loss transfers from a merchant seller to a buyer upon the buyer’s physical receipt of goods. Therefore, neither tender of the goods at the seller’s place of business, nor use of the seller’s truck to deliver the goods are events which transfer risk of loss to the buyer as the merchant seller still retains possession of the goods.

32. (d) The UCC places risk of loss on the breaching party. Since Cey shipped nonconforming goods, it breached the contract and would have risk of loss until the nonconforming goods were accepted by the buyer or until the goods were cured by Cey. Since Deck rejected the goods and Cey did not cure the goods, risk of loss remained with Cey. Shipping terms have no bearing on risk of loss in this situation because the goods did not conform to the contract. Answer(a) is incorrect because Deck would only bear risk of loss if the goods conformed to the contract. Answer(b) is incorrect because the risk of loss was never transferred to Deck since the goods were nonconforming. Answer(c) is incorrect because if the goods were conforming, risk of loss would pass to Deck at Cey’s warehouse based on the shipping terms “FOB Cey’s warehouse.”

33. (b) The parties to the contract may agree as to which party bears risk of loss. In the absence of this, under the UCC, the shipping terms determine who bears risk of loss.

34. (a) In this situation, since Wool is not a merchant seller, the risk of loss passed to Bond on Wool’s tender of delivery. If Wool had been a merchant seller, then the risk of loss would not have passed until the buyer received the goods. Answers(c) and(d) are incorrect because the risk of loss passed when the nonmerchant seller (Wool) tendered delivery of the painting. Answer(b) is incorrect because the risk of loss would not pass at the time the contract was formed since the seller still had possession of the painting and had not attempted to deliver it to the buyer.

35. (b) Common carriers are not liable for losses due to causes deemed acts of God. Although a common carrier may limit its damages to a dollar amount specified in the contract, it is not liable at all in this case. Funston, not the customer, had the risk of loss due to the FOB terms.

36. (b) Statement I is incorrect because a liquidated damages provision is enforced if it is not punitive but amounts to a reasonable estimate of what the loss will be in the event of a breach of contract. If a reasonable estimate of the loss from a breach of contract cannot be estimated with a reasonable degree of certainty, the parties can agree on an amount, but still the amount cannot be punitive. Statement II is correct because damages that are excessive are viewed as punitive.

37. (b) The seller generally discharges his obligation to the buyer by placing conforming goods at the buyer’s disposition and giving the buyer reasonable notice to enable the buyer to take delivery. Answers (a) and (c) are incorrect because the general rule under the UCC is that the place of tender is the seller’s place of business; there is no obligation to deliver the goods to the buyer or to a common carrier. Answer (d) is incorrect because there is no obligation for the seller to set aside goods for inspection prior to delivery; generally the buyer has a right to inspect the goods at the time of delivery, but not prior to delivery.

38. (a) Under the Sales Article of the UCC, the seller has the following remedies against the buyer upon breach: withhold delivery of the goods; stop delivery of the carrier of the goods; resell the goods; recover compensatory and incidental damages; recover the goods from the buyer upon the buyer’s insolvency; cancel the contract. Therefore, answer(a) is correct as the seller has the rights of contract cancellation and damage recovery available to him/her.

39. (a) The UCC gives the seller a choice of many remedies when the buyer breaches the contract involving a sale of goods. These remedies include allowing the seller to resell the goods identified to the contract and to recover the amount that the seller receives that is less than the contract price. Also, once the buyer breaches, the seller may suspend his/her performance and may prevent the carrier from making the delivery of the goods.

40. (b) A seller has the right to resell goods to another if the buyer refuses to accept the goods upon delivery. Answer(a) is incorrect because specific performance is not a remedy available to the seller, since the seller is only receiving money, which is not unique. Baker cannot force Lazur to accept the word processor. Answer(c) is incorrect because Baker has a couple of additional remedies available. Baker can recover the full contract price plus incidental damages if he is unable to resell the identified goods. Alternatively, if the difference between the market value and contract price is inadequate to place Baker in as good a position as performance would have, then Baker can sue for lost profits plus incidental damages. Answer(d) is incorrect because Baker could sue for consequential damages that Lazur had reason to know Baker would incur as a result of Lazur’s breach.

41. (a) By advising Mazur on June 1 that it would not accept or pay for the wheat, Good has engaged in anticipatory repudiation. Anticipatory repudiation occurs when a party renounces the duty to perform the contract before the party’s obligation to perform arises. Anticipatory repudiation discharges the nonrepudiating party (Mazur) from the contract and allows this party to sue for breach immediately. In this situation, Mazur could successfully sue Good for the difference between the resale price and the contract price on June 2. Answer(b) is incorrect because Mazur was discharged from the contract on June 1 and would not have to wait until after June 23 to resell the wheat. Answer(c) is incorrect because Good would only be allowed to retract its anticipatory breach if Mazur had ignored this breach and awaited performance at the appointed date. Answer(d) is incorrect because specific performance is only allowed for unique goods or for other situations in which monetary damages are not appropriate.

42. (c) Pickens has committed an anticipatory breach of contract. Thus, Crocket, as the aggrieved party, has different options. Crocket may treat it as a present breach of contract with the remedies available for breach of contract. One of these remedies is that the aggrieved party (Crocket) may cancel the contract. Another option is that Crocket may wait for a reasonable time to see if Pickens will change his/her mind and still deliver.

43. (b) The buyer has the following remedies against the seller: upon receipt of nonconforming goods, the buyer may reject the goods, accept the goods, or accept any unit and reject the remainder; the buyer has the right to cover (purchase goods elsewhere upon the seller’s breach); the buyer may recover damages (not punitive) for nondelivery of goods or repudiation of the sales contract by the seller; the buyer may recover damages (not punitive) for breach in regard to accepted goods; the buyer may recover goods identified in the contract in possession of the seller upon the seller’s insolvency; the buyer may sue for specific performance when the goods are unique; the buyer has the right of replevin (form of legal action to recover specific goods from the seller which are being withheld from the buyer wrongfully); the buyer can cancel the contract; the buyer has a security interest in the goods after the seller’s breach; the buyer can recover liquidated damages. Punitive damages, however, are not an available remedy in either the common law or the UCC.

44. (d) UCC Section 2-719(3) states that a limitation of damages for personal injury in the case of consumer goods is considered to be unconscionable and thus not allowed. Although limitations of damages for personal injury in the case of nonconsumer goods can be allowed, answer(d) is correct since one limits “personal injury” to the stove which was apparently being used for consumer use in this fact pattern. Answer(a) is incorrect because under the UCC, the spouse, being a member of the household expecting to use the stove, may recover for damages. Answer(b) is incorrect because Larch was notified shortly after the explosion. This notice, however, was not required. Answer(c) is incorrect because even though the disclaimer did not disclaim all liability, it did attempt to disclaim personal injury. This disclaimer for personal injuries, however, is not allowed for the reasons mentioned above. Answer(d) is chosen as being more specific than answer(c).

45. (d) Either party in a sales contract under the Sales Article of the UCC may demand adequate assurance of performance when reasonable grounds for insecurity exist with respect to the performance of the other party. Refusal to give written assurance will release the other party from all obligations from the sales contract. Answer(a) is incorrect because the buyer has assumed the risk of loss. Answer(b) is incorrect because a seller may substitute another reasonable delivery method if the method of delivery specified in the contract has been made impracticable. A seller may recover damages based on a buyer’s repudiation of the agreement, but here the repudiation has been retracted and the obligations of buyer and seller remain intact.

46. (c) The Sales Article of the UCC provides that a buyer has the right to reject goods which are not in conformity with the terms of contract between seller and buyer. The buyer also has the option to accept nonconforming goods and recover damages resulting from the nonconformity. The UCC allows the buyer to inspect the goods before payment except when they are shipped COD. When goods are shipped COD, the buyer’s payment for the goods is required for delivery.

47. (b) There are two remedites for fraud under the UCC Sales Article: (1) the plaintiff may affirm the agreement and sue for damages under the tort of deceit, or (2) the plaintiff may rescind the contract and sue for damages resulting from the fraud. Answer (a) is incorrect because the plaintiff must return any consideration received from the other party when the contract is rescinded. Answer (c) is incorrect because although punitive damages are allowed in fraud actions because they are intentional torts, they do not require physical injuries. Answer (d) is incorrect because without reliance by the plaintiff on the misrepresentation, there is no fraud, and therefore, the plaintiff may not rescind the contract.

48. (c) The statute of limitations for the sale of goods is generally four years; however, the parties may agree to reduce the statute to a period of not less than one year. Therefore, Sklar will lose because the clause providing that the statute of limitations would be limited to eighteen months is enforceable, and the action was not brought within the required time period. Answer(b) is incorrect because a breach of warranty occurs upon the tender of delivery, not upon the discovery of the defect, and the statute begins running at the time the breach occurs. Answer(d) is incorrect because the statute is eighteen months as outlined in the contract.

Simulations

Task-Based Simulation 1

image

Situation

Angler Corp., a food distributor, is involved in the following disputes:

  • On September 8, Angler shipped the wrong grade of tuna to Mason Restaurants, Inc. under a contract that stated as follows: “FOB Angler’s loading dock.” During shipment, the tuna was destroyed in an accident involving the common carrier’s truck. Mason has refused to pay for the tuna, claiming the risk of loss belonged to Angler at the time of the accident.
  • On October 3, Angler shipped 100 bushels of peaches to Classic Foods, Inc., a retail grocer. Because of a delay in shipping, the peaches rotted. Classic elected to reject the peaches and notified Angler of this decision. Angler asked Classic to return the peaches at Angler’s expense. Classic refused the request, claiming it had no obligation to do so.
  • On October 23, Angler orally contracted to sell Regal Fast-Food 1,500 pounds of hamburger meat for $1,500. Delivery was to be made on October 31. On October 29, after Angler had shipped the hamburger meat to Regal, Regal sent Angler the following signed correspondence:
“We are not going to need the 1,500 pounds of meat we ordered on October 23. Don’t ship.”
Regal rejected the shipment and claimed it is not obligated to purchase the hamburger meat because there is no written contract between Angler and Regal.
Determine whether each of the numbered legal conclusions is Correct or Incorrect.
  Correct Incorrect
1. When the accident happened, the risk of loss belonged to Angler. image image
2. If Angler had shipped the correct grade of tuna to Mason, the risk of loss would have been Angler’s at time of the accident. image image
3. The contract between Angler and Mason was an FOB destination point contract. image image
4. Angler had title to the tuna at time of the accident since Angler shipped nonconforming goods. image image
5. Classic is required to return the peaches at Angler’s expense per Angler’s instructions. image image
6. Classic may throw the peaches away because they were rotted. image image
7. Since Classic elected to reject the rotted peaches, Classic may not also sue for damages. image image
8. Regal is not obligated to purchase the hamburger meat because there was no written contract between Angler and Regal. image image
9. The Uniform Commercial Code applies to the contract between Angler and Regal. image image
10. Regal’s correspondence to Angler, dated October 29, satisfies the appropriate Statute of Frauds. image image
11. Angler should keep the hamburger until Regal finally accepts it and sue Regal for $1,500. image image
12. Assuming that all of the original facts are the same except that Regal never sent Angler the correspondence dated October 29, then Angler may hold Regal in breach of contract. image image
13. Assuming that all of the original facts are the same except that the contract was for $450, then Angler may hold Regal in breach of contract. image image
14. Assume that all of the original facts are the same except that Regal never sent Angler the correspondence and Angler shipped to Regal 800 pounds of the hamburger on October 29. Regal accepted the 800 pounds. Regal, then, on October 31 orally rejected the shipment for the remaining 700 pounds. Under these facts, the contract is enforceable against Regal for the 800 pounds but not the full 1,500 pounds. image image
15. Under the same facts found in 14. above, the contract is enforceable against Regal for the full 1,500 pounds. image image

Task-Based Simulation 2

image

Situation

On February 1, Grand Corp., a manufacturer of custom cabinets, contracted in writing with Axle Co., a kitchen contractor, to sell Axle 100 unique, custom-designed, kitchen cabinets for $250,000. Axle had contracted to install the cabinets in a luxury condominium complex. The contract provided that the cabinets were to be ready for delivery by April 15 and were to be shipped FOB seller’s loading dock. On April 15, Grand had eighty-five cabinets complete and delivered them, together with fifteen standard cabinets, to the trucking company for delivery to Axle. Grand faxed Axle a copy of the shipping invoice, listing the fifteen standard cabinets. On May 1, before reaching Axle, the truck was involved in a collision and all the cabinets were damaged beyond repair.

Items 1 through 5 refer to the above fact pattern. For each item, determine whether (A), (B), or (C) is correct.

image

Simulation Solutions

Task-Based Simulation 1

image
  Correct Incorrect
1. When the accident happened, the risk of loss belonged to Angler. image image
2. If Angler had shipped the correct grade of tuna to Mason, the risk of loss would have been Angler’s at time of the accident. image image
3. The contract between Angler and Mason was an FOB destination point contract. image image
4. Angler had title to the tuna at time of the accident since Angler shipped nonconforming goods. image image
5. Classic is required to return the peaches at Angler’s expense per Angler’s instructions. image image
6. Classic may throw the peaches away because they were rotted. image image
7. Since Classic elected to reject the rotted peaches, Classic may not also sue for damages. image image
8. Regal is not obligated to purchase the hamburger meat because there was no written contract between Angler and Regal. image image
9. The Uniform Commercial Code applies to the contract between Angler and Regal. image image
10. Regal’s correspondence to Angler, dated October 29, satisfies the appropriate Statute of Frauds. image image
11. Angler should keep the hamburger until Regal finally accepts it and sue Regal for $1,500. image image
12. Assuming that all of the original facts are the same except that Regal never sent Angler the correspondence dated October 29, then Angler may hold Regal in breach of contract. image image
13. Assuming that all of the original facts are the same except that the contract was for $450, then Angler may hold Regal in breach of contract. image image
14. Assume that all of the original facts are the same except that Regal never sent Angler the correspondence and Angler shipped to Regal 800 pounds of the hamburger on October 29. Regal accepted the 800 pounds. Regal, then, on October 31 orally rejected the shipment for the remaining 700 pounds. Under these facts, the contract is enforceable against Regal for the 800 pounds but not the full 1,500 pounds. image image
15. Under the same facts found in 14. above, the contract is enforceable against Regal for the full 1,500 pounds. image image

Explanations

1. (C) Angler breached the contract by shipping nonconforming goods to Mason. Therefore, Angler retains the risk of loss until it cures or until Mason accepts the goods despite the nonconformity.

2. (I) This was an FOB shipping point contract so that the risk of loss would have passed over to the buyer upon delivery to the carrier.

3. (I) Because the terms were FOB the seller’s loading dock, it was an FOB shipping point contract.

4. (I) Title and risk of loss do not necessarily pass to a buyer at the same time. In this case, risk of loss remained with the seller because of the shipment of nonconforming goods. However, title passed under the original terms despite the breach of contract.

5. (C) Classic is obligated to follow any reasonable instructions of the seller as a merchant who rejects goods, even nonconforming, under a contract.

6. (I) Classic must follow the reasonable instructions given by Angler to return the peaches at Angler’s expense.

7. (I) Classic may also sue for any damages that were caused by the delay in shipping.

8. (I) Although the contract must be evidenced by a writing because it involved a sale of goods for more than $500, the correspondence that Regal sent to Angler on October 29 satisfies the writing requirement under the UCC Statute of Frauds. It indicated that a contract had been made. It was signed by Regal, the party to be charged, and it stated the quantity. The price was not needed in the correspondence.

9. (C) The Uniform Commercial Code applies because the contract was for a sale of goods (i.e., hamburger meat).

10. (C) The correspondence satisfies the UCC Statute of Frauds which does not require that all terms be in writing.

11. (I) Angler should resort to an appropriate remedy such as reselling the hamburger to someone else in a commercially reasonable fashion. If Angler gets less than the original contract price, it may recover the difference from Regal.

12. (I) Since there was no writing to evidence the contract for $1,500, it is not enforceable.

13. (C) The contract need not be in writing because it was for less than $500.

14. (C) Since Angler shipped and Regal accepted a portion of the goods, the oral contract is enforceable up to the amount shipped and accepted. This is one of the exceptions in the UCC Statute of Frauds.

15. (I) The exception in the UCC Statute of Frauds allows the oral contract to be enforced up to the amount delivered and accepted or paid for.

Task-Based Simulation 2

image
image

Explanations

1. (A) The terms of the contract were “FOB seller’s loading dock” which is a shipment contract. Answer(B) is incorrect because a destination contract would state terms meaning FOB buyer’s location. Answer(C) is incorrect because a consignment is treated as a sale or return. That is, the owner of the goods delivers them to another party to attempt to sell them. If this other party, known as the consignee, does not sell the goods, they are returned. Such is not the case in this fact pattern.

2. (C) Risk of loss would normally pass to the buyer, Axle Co., under this shipment contract. However, since the seller, Grand, breached the contract, risk of loss remains with Grand. Since the cabinets are “custom designed, kitchen cabinets” for a luxury condominium complex, they would need to match. Therefore, the 100 units could be construed as a commercial unit and the risk of loss for the entire 100 cabinets remained with Grand. Answer(A) is incorrect because the 100 cabinets were a commercial unit and thus the risk of loss of the entire commercial unit remained with Grand. Answer(B) is incorrect because even though the terms were “FOB seller’s loading dock,” the risk of loss remained with the seller, Grand, because of Grand’s breach of contract.

3. (B) The contract between Grand and Axle was voidable because Axle may at its option choose to accept or reject all or part of the cabinets. Answer(C) is incorrect because if the contract were void, neither party would have the option of remaining in the contract. Answer(A) is incorrect because under the UCC, if the delivery date is not stated, the time becomes within a reasonable time.

4. (A) Once Grand ships nonconforming goods, a breach of contract has occurred. Answer(B) is incorrect because the breach has occurred even without Axle needing to reject the shipment. Axle then has the right to accept all, part, or none of the shipment. Answer(C) is incorrect because the shipment of nonconforming goods acts as a breach rather than a counteroffer.

5. (A) Since the cabinets are unique and custom-designed, specific performance is allowed if Axle so chooses. Answer(B) is incorrect because Axle is not required to cover, especially because the cabinets are unique. Answer(C) is incorrect because punitive damages are generally not allowed for a breach of contract even if the breach is intentional.

..................Content has been hidden....................

You can't read the all page of ebook, please click here login for view all page.
Reset
13.59.197.213